Maria is buying new carpet for her bedroom .Her bedroom is in the shape of a square and the length of each side is 12 feet write and simplify an exponential express to find how much carpet she needs.

Answers

Answer 1

Answer:

well just do area, and since it's the same in each side 12×4= 144


Related Questions

How do you graph y=2/3x-4

Answers

━━━━━━━☆☆━━━━━━━

▹ Answer

You can use a graphing calculator.

▹ Step-by-Step Explanation

Attached is a screenshot.

Hope this helps!

CloutAnswers ❁

━━━━━━━☆☆━━━━━━━

Answer:

See explanation and picture attached

Step-by-step explanation:

We can break down this expression into it's core components:

Since the constant here is -4, the y intercept is -4.

Since the value we are multiplying x by is [tex]\frac{2}{3}[/tex], the slope is  [tex]\frac{2}{3}[/tex]. This means for every time we go horizontal 3 units, the line increases by 2.

The graph is attached.

Hope this helped!

E
What is the value of x in the equation 3x.. by y 18, when y27

Answers

Answer:

x = 15

Step-by-step explanation:

We need to find the value of x in the equation 3x – y = 18 when y = 27.

To find the value of x, put y = 27 in the above equation.

So,

3x - 27 = 18

3x = 45

x = 15

So, the value of x is 15.

which one is irrational?

Answers

Answer: A, B, D are irrational

Basically everything but choice C

==========================================

Explanation:

sqrt is shorthand for square root

sqrt(4) = 2 = 2/1 showing that sqrt(4) is rational. We can write it as a fraction of two whole numbers, where 0 is not in the denominator.

-------

In contrast, we cannot write sqrt(2), sqrt(3), or sqrt(5) as a fraction of two whole numbers. Using your calculator, note how

sqrt(2) = 1.4142135623731

sqrt(3) = 1.73205080756888

sqrt(5) = 2.23606797749979

all of those decimal expansions go on forever without any pattern, which is a sign that those numbers are irrational. If they were rational, then a pattern would repeat at some point or the decimals would terminate at some point.

Answer:

a, b, d are irrational

Step-by-step explanation:

root 2 = 0.414.....

root 3 = 0.732.....

root 5 = 2.236.....

Hope this helps.....

Pls mark my ans as brainliest

If u mark my ans as brainliest u will get 3 extra points

Do the ratios 2/3 and 12/18 form a proportion?
yes
no​

Answers

Answer:

Yes

Step-by-step explanation:

Because 12/18 = 2/3..(cancel 12 and 18 by 6)

Answer:

yes

Step-by-step explanation:

2x6=12

3x6=18

6 is the multiplying number

( the 2 equations are the same amount )

A debt of $12,000 with interest at 5% compounded monthly is to be repaid by equal payments at the end of each year for three years and nine months. What is the term of repayment? None 12 months 3.9 years 3.75 years

Answers

Answer:

  3.75 years

Step-by-step explanation:

If the debt is to be paid in 3 years, 9 months, then the term of the loan is ...

  3 9/12 = 3 3/4 = 3.75 . . . years

The three-dimensional figure shown consists of a cylinder and a right circular cone. The radius of the base is 10 centimeters. The height of the cylinder is 16 centimeters, and the total height of the figure is 28 centimeters. The slant height of the cone is 13 centimeters. Which choice is the best approximation of the surface area of the figure? Use 3.14 to approximate pi.

Answers

Answer:

2,041 square centimeters

Step-by-step explanation:

surface area = (2 × π × r × h) + ((π × r) × (r+ (√(c² + r²))))+(π × r²)

where,

cylinder  base radius (r) = 10  cm

height of cylinder (h) = 16 cm

total height = 28 cm

cone height (c) = total height - height of cylinder = 28 - 16 = 12cm

π = 3.14

surface area = (2 × 3.14 × 10 × 16) + ((3.14 × 10) × (10+ (√(12² + 10²))))+(3.14 × 10²)

surface area = 1004.8 + (31.4 * 25.6) + 314

surface area = 2122.64 cm²

therefore the approximate surface area given is 2,041 square centimeters

Answer ASAP, Will give brainliest!!

Answers

Answer:

First. 115°

Second. 65°

Third. 65°

Fourth. 7

Fifth. 425.25

First

angle DAB = angle ADC (since this is an isosceles trapezoid)

Second

In a trapezoid adjacent angle are supplmentary (that is their sum is 180°)

180-115 is 65°

Third

(Same reason as second)

Fourth

The side 3x+4 is same as the opposite side

So 3x + 4 = 25

on solving you get x = 7 in

Fifth

[tex]area \: = \frac{1}{2} \times length \: of \: the \: perpendicular \: (b1 + b2)[/tex]

area = 1/2 × 13.5 (20+43)

area = 1/2 × 13.5 × 63

Thus area is 425.25

Answer:

Step-by-step explanation:

1)As ABCD is isosceles trapezium,

∠ADC= ∠DAB

∠ADC = 115°

2) AD //BC

∠ADC + ∠DCB = 180°   {co interior angles}

115 + ∠DCB = 180

 ∠DCB = 180 - 115

 ∠DCB = 65°

3) As ABCD is isosceles trapezium,

∠CBA = ∠DCB

∠CBA = 65°

4) As ABCD is isosceles trapezium, non parallel sides are congruent.

AB = DC

3x + 4 = 25 in

3x = 25 - 4

3x = 21

x = 21/3

x = 7 in

5) height = 13.5 in

a= 43 in

b= 20 in

Area of trapezium = [tex]\frac{(a+b)*h}{2}\\[/tex]

                   [tex]= \frac{(43 +20)*13.5}{2}\\\\=\frac{63*13.5}{2}\\\\\\= 425.25 in^{2}[/tex]

Given that ΔABC is a right triangle with the right angle at C, which of the following is true?

1. tan A = 1/(tan B)
2. tan A = sin B
3. cos A = 1/(cos B)
4. sin B = 1/(sin A)

Answers

Answer:

1. tan A = 1/(tan B)

Step-by-step explanation:

By definition,

tangent A = opposite / adjacent = a / b

and

tangent B = opposite / adjacent = b / a

Therefore tangent A = a/b = 1/tan(B)

Answer: Tan a=tan b

I belive

Step-by-step explanation:

A line passes through point (4,-3) and has a slope of 5/4. Write an equation in Ax + By = C

Answers

Answer:

The answer is

5x - 4y = 32

Step-by-step explanation:

To write an equation of a line using a point and slope use the formula

y - y1 = m(x - x1)

where

m is the slope

(x1 , y1) is the point

So we have

Equation of the line using point (4 , -3) and slope 5/4 is

[tex]y + 3 = \frac{5}{4} (x - 4)[/tex]

Multiply through by 4

4y + 12 = 5(x - 4)

4y + 12 = 5x - 20

5x - 4y = 20 + 12

The final answer is

5x - 4y = 32

Hope this helps you

A driver of a car stopped at a gas station to fill up his gas tank. He looked at his watch, and the time read exactly 3:40 p.m. At this time, he started pumping gas into the tank. At exactly 3:44, the tank was full and he noticed that he had pumped 6 gallons.

Answers

Answer:

3.625 gpm

Step-by-step explanation:

Plz Help I Will Mark Brainliest If Right!!!!!!!!!!!!!!!!!!!!!!!
Determine the domain of the function.

f as a function of x is equal to the square root of one minus x.

A). All real numbers
B). x > 1
C). x ≤ 1
D). All real numbers except 1

Answers

Hey There!!~

Your best answer choice is B). x > 1.

Good Luck!!

What is the image point of (-5,9) after a translation left 1 unit and down 1 unit?

Answers

Answer: (-6,8)

Step-by-step explanation:

Translation is a rigid motion inn which every point of the figure moved in the same direction and for the same distance

Translation rules are

Left c units : [tex](x,y)\to(x-c,y)[/tex]

Down c units : [tex](x,y)\to(x,y-c)[/tex]

The image point of (-5,9) after a translation left 1 unit and down 1 unit will be:

[tex](-5,9)\to(-5-1,9-1)=(-6,8)[/tex]

Hence, the image point is (-6,8).

is 1 whole 1 by 3 considered as an integer??

Answers

Answer:

No it's not.

Step-by-step explanation:

[tex]1 \frac{1}{3} = \frac{4}{3} = 1.333333[/tex]

Integers are set of numbers consisting

Whole numberNatural numberNegative numbers

It doesn't consist

Fractions &Decimals

Hope this helps ;) ❤❤❤

Answer:

[tex]\boxed{\sf No}[/tex]

Step-by-step explanation:

[tex]\displaystyle 1 \frac{1}{3} =1.3333333...[/tex]

Integers are whole numbers that can be positive or negative. Integers do not include fractions and decimals.

1. At the end of one school day a teacher had 17 crayons left. The teacher remembered
giving out 14 crayons in the morning, getting 12 crayons back at recess, and giving out
11 crayons after lunch. How many crayons did the teacher have at the start of the
day?

Answers

Answer:

30 crayons

Step-by-step explanation:

Let x be the number of crayons he started with

gave out 14 crayons

x-14

Got 12 back

x-14+12

Gave out 11 after lunch

x-14+12 -11

This equals 17

x-14+12 -11 =17

Combine like terms

x-13 = 17

Add 13 to each side

x -13+13 =17+13

x = 30

Answer: 30

Step-by-step explanation:

For this problem work backwards. Start from 17 and add 14. You should get 31. Then subtract 12, which equals 19. Finally add 11 to 19, which equals 30. Basically you are doing the inverse operation to get your answer. Hope this helps!

a rectangle is 12 in wide and 18 in tall.if it is reduce to a height of 3 inches, then how wide will it be?

Answers

Answer:

2 in

Step-by-step explanation:

18/3=6 , 6 is the scale factor

12/6=2

Answer:

width= 2

Step-by-step explanation:

18 inches is the original height and we are now reducing that to 3 inches.

In order to do that, we have to divide 18 by 3 which equals 6.

Next, take the width of the rectangle, which is twelve and divide it by the scale factor of 6 which  equals 2.

Your final answers should be: width= 2

Find the next three terms in the geometric sequence.

Answers

Answer: D

Step-by-step explanation:

The common difference is  -2/3  so using the last term which is -8/27 multiply it by -2/3 to find the next terms.

[tex]-\frac{8}{27} * -\frac{2}{3}[/tex] = [tex]\frac{16}{81}[/tex]    

[tex]\frac{16}{81} * -\frac{2}{3} = -\frac{31}{243}[/tex]  

[tex]-\frac{32}{243} * -\frac{2}{3} = \frac{64}{729}[/tex]  

Write the expression 12-2 in simplest form.

Answers

Answer:

convert into a whole number 6

A principal of $2600is invested at 6.75% interest, compounded annually. How much will the investment be worth after 14 years

Answers

Answer:

$6488.19

Step-by-step explanation:

To solve this problem we use the compounded interest formula:

[tex]amount = principal(1 + (r \n))^({n}{t})[/tex]

a = $2600(1+(0.0675/1))¹*¹⁴

a = $6488.19

Each lap around a park is 1 1⁄5 miles. Kellyn plans to jog at least 7 1⁄2 miles at the park without doing partial laps. How many laps must Kellyn jog to meet her goal?

Answers

Answer:

25/4 laps or (6.25 laps)

Step-by-step explanation:

1 lap = 1 1/5 miles

kellyn plans to jog 7 1/2 miles

                                                      1  lap

number of laps = 7 1/2 miles x  --------------   = 25/4 laps or (6.25 laps)

                                                    1 1/5 miles

inscribed angles. help asap!​

Answers

Answer:

20°

Step-by-step explanation:

The measure of the inscribed angle is equal to the half of the arc it sees

Since AC is the diameter the measure of arc ABC is 180°

and since A sees arc BC and C sees the arc AB

A< + C< = 90° so angle C = 20°

What is the value of x in the equation 3x-4y=65, when y =4 will give brainliest

Answers

Hello!

Answer:

[tex]\huge\boxed{x = 27}[/tex]

Given:

3x - 4y = 65 where y = 4;

Substitute in 4 for "y":

3x - 4(4) = 65

Simplify:

3x - 16 = 65

Add 16 to both sides:

3x - 16 + 16 = 65 + 16

3x = 81

Divide both sides by 3:

3x/3 = 81/3

x = 27.

Hope this helped you! :)

Answer:

x=27

Step-by-step explanation:

3x-4y=65

Let y=4

3x - 4(4) = 54

3x -16 = 65

Add 16 to each side

3x -16+16 = 65+16

3x = 81

Divide each side by 3

3x/3 =81/3

x =27

Nina is training for a marathon. She can run 4 1/2​ kilometers in 1/3​ of an hour. At this pace, how many kilometers can Nina run in 1 hour?

Answers

Answer:

Nina can run:

13 1/2 km in 1 hour

Step-by-step explanation:

4 1/2 = 4 + 1/2 = 8/2 + 1/2 = 9/2

proportions:

9/2 hours ⇔  1/3 hour

N hours ⇔ 1 hour

N = (9/2)*1 / (1/3)

N = (9/2) / (1/3)

N = (9*3) / (2*1)

N = 27/2

27/2 = 26/2 + 1/2 = 13 + 1/2 = 13 1/2

Nina can run:

13 1/2 km/h

13 1/2 km in 1 hour

Nina can run [tex]13\frac{1}{2}[/tex] km in an hour

The distance Nina can run in an hour can be determined by dividing the distance she can run in 1/3 of an hour by 1/3

Distance Nina can run in an hour = distance run ÷ [tex]\frac{1}{3}[/tex]

[tex]4\frac{1}{2}[/tex] ÷ [tex]\frac{1}{3}[/tex]

Convert the mixed fraction to an improper fraction [tex]\frac{9}{2}[/tex] × 3 = [tex]\frac{27}{2}[/tex]

Convert the improper fraction back to an mixed fraction = [tex]13\frac{1}{2}[/tex] km

To learn more about fractions, please check:

https://brainly.com/question/21449807?referrer=searchResults

if a man works 400km in 6 minutes.How long will he work in 9 minutes​

Answers

Answer:

600 km

Step-by-step explanation:

400 km  =       x    

  6 min         9 min  

cross multiply:

6x = 400 ( 9)

x = 3600 / 6

x = 600 km

One researcher wishes to estimate the mean number of hours that high school students spend watching TV on a weekday. A margin of error of 0.28 hour is desired. Past studies suggest that a population standard deviation of hours is reasonable. Estimate the minimum sample size required to estimate the population mean with the stated accuracy.

Answers

Complete question:

One researcher wishes to estimate the mean number of hours that high school students spend watching TV on a weekday. A margin of error of 0.28 hours is desired. Past studies suggest that a population standard deviation of 1.5 hours is reasonable. Estimate the minimum sample size required to estimate the population mean with the stated accuracy.

Answer:

111 students

Step-by-step explanation:

Given the following :

Margin of Error (E) = 0.28

Population standard deviation (sd) = 1.5

Recall:

Margin of Error(E) = Z * (sd/√n)

Taking a confidence interval of 95%

The Z value at a 95% confidence interval is 1.96

Plugging our values, we have :

Margin of Error(E) = Z * (sd/√n)

0.28 = 1.96 * (1.5/√n)

0.28 = 2.94 / √n

√n × 0.28 = 2.94

√n = 2.94 / 0.28

√n = 10.5

Square both sides to obtain n

n = 10.5^2

n = 110.25

Hey there please help me with this question

Answers

Answer:

see explanation

Step-by-step explanation:

sum the parts of the ratio, 2 + 1 = 3 parts , thus

81 cm² ÷ 3 = 27 cm² ← value of 1 part of the ratio

2 parts = 2 × 27 = 54 cm²

Area of A = 54 cm² and area of B = 27 cm²

The side of the original square = [tex]\sqrt{81}[/tex] = 9 cm

The width of both rectangles is 9 cm ( width remains unchanged after cut )

Thus

Rectangle A

9 × length = 54 ( divide both sides by 9 )

length = 6 cm

Rectangle B

9 × length = 27 ( divide both sides by 9 )

length = 3 cm

Rectangle A → length = 6 cm, width = 9 cm

Rectangle B → length = 3 cm , width = 9 cm

Answer:

Rectangle A                 Rectangle B

length = 9 cm                length = 9 cm

width = 6 cm                  width = 3 cm

Step-by-step explanation:

Area of square At = 81 cm²

Square is cut into two pieces = A + B

The ration of area A to B = 2:1

Find

Rect A        Rect B

length         length

width           width

---------------------------------

first, get the side of the square = A = s²

81 = s²,      

s = √81      

s = 9 cm

since the ratio is 2:1, therefore the side can be divided into 3

9 ÷ 3 = 3 cm ----- take note of this to get the Width

Rectangle A

L = 9 cm (which is the s = 9 cm)

W = 3 cm (2 ratio) = 6 cm

Rectangle B

L = 9 cm  (which is the s = 9 cm)

W = 3 cm (1 ratio) = 3 cm

Proof:

At = A + B

81 = (9x6) + (9x3)

81 = 54 + 27

81 = 81  ----- OK

If area of a rhombus is 336 cm and one of its diagonal is 14 cm, find its perimeter.

Answers

Answer:

The perimeter of the Rhombus is 100 cm

Step-by-step explanation:

First of all, we will need to find the length of the other diagonal.

let’s call the diagonals p and q

Mathematically, the area of the Rhombus is;

pq/2 = Area of Rhombus.

Let’s call the missing diagonal p

So;

(p * 14)/2 = 336

14p = 672

p = 672/14

p = 48 cm

Now, we can find the perimeter of the Rhombus using these diagonals.

Mathematically;

P = 2 √(p^2 + q^2)

Substituting these values, we have;

P = 2 √(14)^2 + (48^2)

P = 2 √(2500)

P = 2 * 50

P = 100 cm

The perimeter of the rhombus is the sum of its side lengths

The perimeter of the Rhombus is 100 cm

The length of one of its diagonal is given as:

[tex]p= 14[/tex]

And the area is given as:

[tex]A = 336[/tex]

Assume the other diagonal is q.

The area of the rhombus is represented as:

[tex]A = \frac{pq}2[/tex]

So, we have:

[tex]336 = \frac{14q}2[/tex]

This gives

[tex]336 = 7q[/tex]

Divide both sides by 7

[tex]48 = q[/tex]

Rewrite as:

[tex]q = 48[/tex]

The perimeter (P) of the rhombus is calculated as:

[tex]P =2\sqrt{p^2 + q^2[/tex]

So, we have:

[tex]P =2\sqrt{48^2 + 14^2[/tex]

Evaluate the squares

[tex]P =2\sqrt{2500[/tex]

Take positive root of 2500

[tex]P =2 \times 50[/tex]

[tex]P =100[/tex]

Hence, the perimeter of the Rhombus is 100 cm

Read more about areas and perimeters at:

https://brainly.com/question/14137384

Evaluate without actual multiplication 1) 95x96 2)103x107

Answers

Answer:

:

"(100 + 3) (100 + 7)

Now, by using identity

(x + a) (x + b) = x² + (a+b)*x + ab

So,

x = 100 , a = 3 , b = 7

= (100)² + (3+7)*100 + (3*7)

= 10000 + 1000 + 21

= 11021

.

(110 - 7) (110 - 3)

by using identity

(x + a) (x + b) = x² + (a+b)*x + ab

So,

x = 100 , a = (-7) , b = (-3)

= (110)² + { (-7) + (-3) }*110 + {(-7)*(-3)}

= 12100 + (-10)*110 + 21

= 21200 - 1100 + 21

= 11021

.

➖➖➖➖➖➖➖➖➖➖

.

(90 + 5) (90 + 6)

by using identity

(x + a) (x + b) = x² + (a+b)*x + ab

So,

x = 90 , a = 5 , b = 6

= (90)² + (5+6)*90 + (5*6)

= 8100 + 990 + 30

= 9120

.

(100 - 5) (100 - 4)

by using identity

(x + a) (x + b) = x² + (a+b)*x + ab

So,

x = 100 , a = (-5) , b = (-4)

= (100)² + { (-5) + (-4) }*100 + 20

= 10000 + (-9)*100 + 20

= 10000 - 9000 + 20

= 10020 - 900

= 9120

.

➖➖➖➖➖➖➖➖➖➖

.

(100 + 4) (100 - 4)

by using identity

(x + a) (x + b) = x² + (a+b)*x + ab

So,

x = 100 , a = 4 , b = (-4)

= (100)² + { 4 + (-4) }*100 + 4*(-4)

= 10000 + (4 - 4)*100 - 16

= 10000 + 0*100 - 16

= 10000 - 16

= 9984

.

(90 + 14) (90 + 6)

by using identity

(x + a) (x + b) = x² + (a+b)*x + ab

So,

x = 90 , a = 14 , b = 6

= (90)² + (14 + 6)*90 + (14*6)

= 8100 + 20*90 + 84

= 8100 + 1800 + 84

= 9984"

This answer was in another question

This answer was given by BloomingBud

Step-by-step explanation:

Answer:

1) 9120 2) 11021

Step-by-step explanation:

95 * 96 = (100-5)(100-4) = 10000 - 500 - 400 + 20 = 9120

103 * 107 = (100+3)(100+7) = 10000 + 300 + 700 + 21 = 11021

Determine if the ordered pair (6, 4) is a solution to the inequality

Answers

Answer:

[tex]\Large \boxed{\mathrm{Option \ D}}[/tex]

Step-by-step explanation:

(6, 4)

x = 6 and y = 4

y > -1/2x + 7

Plug in the values to check if it is true.

4 > -1/2(6) + 7

4 > -3 + 7

4 > 4

This statement is false.

(6, 4) lies on the line.

The fraction subtracted from 5/3 to get 1 is_____​

Answers

Answer:

2/3

Step-by-step explanation:

I am not sure

Answer:

2/3

Step-by-step explanation:

[tex]Let \:the \: unknown \: fraction \: be \: x\\\\\frac{5}{3} -x = 1\\\\\frac{5}{3}-x=1\\\\\mathrm{Subtract\:}\frac{5}{3}\mathrm{\:from\:both\:sides}\\\\\frac{5}{3}-x-\frac{5}{3}=1-\frac{5}{3}\\\\\frac{5}{3}-x-\frac{5}{3}=-x\\\\1-\frac{5}{3}=-\frac{2}{3}\\-x=-\frac{2}{3}\\\\x=\frac{2}{3}\\[/tex]

243 as a power of 3

Answers

Answer:

243 as a power of 3

= 3^5

=243

Other Questions
Name a real world context to describe the sums of rational numbers. Which rule describes this transformation? (Zoom in to see it clearly) Cual es la diferencia entre las palabras destacadas que tienen tilde y las que no la tienen? A certain light bulb consumes 200J of electrical energy per second, but only emits 25J of light energy per second. Calculate the efficiency of this bulb. In the course of selling a home to a buyer, a broker told the buyer that the home's foundation was "solid as a rock", when he knew for a fact that it was slowly sinking into the landfill on which it was built. In this situation, the broker's conduct would be BEST defined as a Find f(x) and g(x) so the function can be expressed as y = f(g(x)). (1 point) [tex]y=\frac{7}{x^{2} } +10[/tex] Testbank Question 47 Consider the molecular orbital model of benzene. In the ground state how many molecular orbitals are filled with electrons? Compare maps of the world in ancient times with current political maps. Use the maps below to answer the following question. Compare the two maps. Which geographical feature formed the western boundary of the Plains Native American cultural region? A) The Appalachian Mountains B) The Rocky Mountains C) The Mississippi River D) The Pacific Ocean The south defeat, 1865 2. What does the author mean by "The press is now muzzled- and all news comes to us through thesieve of Yankee eyes."?Please help due at 8:00pm 2NH3 N2 + 3H2 If 2.22 moles of ammonia (NH3) decomposes according to the reaction shown, how many moles of hydrogen (H2) are formed? A) 2.22 moles of H2 B) 1.11 moles of H2 C) 3.33 moles of H2 D) 6.66 moles of H2 The graph of f(x) = xx is reflected across the x-axis and then across the y-axis to create the graph of function g(x).Which statements about the functions f(x) and g(x) are true? Check all that apply.The functions have the same range.The functions have the same domains.The only value that is in the domains of both functions is 0.There are no values that are in the ranges of both functions.The domain of g(x) is all values greater than or equal to 0.The range of g(x) is all values less than or equal to 0. The length of a rectangle is three times its width. If the perimeter of the rectangle is 160 cm, what are the dimensions of this rectangle? In which type of economic activity is dairy farming included? Adam published his personal opinion about the current condition of politics stating,"Though the government is unable to fulfill the demands of the public,they demand votes;they really are beggars on streets." Can politicians sue him for defamation under tort?A) No,because as a general rule the material in a newspaper is totally exempted.B) Yes,because defamation in a newspaper is always a tort.C) Yes,because Adam wrote even though he did not have authority.D) No,because statements of pure opinion do not amount to defamation. Does the amount of methanol increase, decrease, or remain the same when an equilibrium mixture of reactants and products is subjected to the following changes? a. the catalyst is removedb. the temp is increasedc. the volume is decreasedd. helium is addede. CO is added **Yoxelt buys 4 1/2 gallons of soda. One-fourth of the soda he bought was Pepsi and the rest was Sprite. How many gallons of Pepsi did Yoxelt buy? Show all work below. Decide all proper subsets of A { 8 ,7 ,6 ,5 ,4 ,3 ,2 ,1} = A 1- { 4 ,3 ,2 ,1} 2- { } 3- { 9 ,8 ,7 } 4- { 11 ,2} 5- { 5 } If the price that determined where marginal revenue equaled marginal cost were below the bottom of the average variable cost curve, then the profit-maximizing, monopolistically competitive firm would which best describes bacterium? Question 5Calculate the kinetic energy of a car (m - 800 kg) moving at 15 m/s. Write your answer to the nearest whole number in the blank spaceprovided. Only write the numerical value of the answer without units. Do not leave any space in between numbers.Answer: Joules